Sums of Reciprocals of Infinite Subsets of Primes

In summary, the conversation discusses the conjecture that if X is an infinite subset of the prime numbers, then the series of the reciprocals of the elements in X either diverges or is irrational. Some examples are given where the series diverges or converges, but it is not clear how to prove the general case. One approach involves using the fact that the primes share no common factors, but it is not rigorous. Another approach uses the density of the primes to construct a subsequence that converges to a given positive real number. However, it is unclear if this method can be used to prove the conjecture for all
  • #1
Dragonfall
1,030
4
Can someone confirm/disprove the following:

If [tex]X\subset\mathbb{P}[/tex] is infinite, then [tex]\sum_{n\in X}\frac{1}{n}[/tex] diverges or is irrational.
 
Mathematics news on Phys.org
  • #2
Some of these series diverge (eg, X=P), and some converge (eg, pick the nth prime in the list to be larger than 2^n). I don't see how you could prove such a a general infinite sum to be irrational.
 
  • #3
It's nowhere near rigorous, but the irrationality part probably has to do with the fact that since they're all primes, the fractions share no common factors (like 1/2 and 1/4 would... if you get what I mean). This means that as you group them, you would need increasingly larger decimals

Take
(p1+p2...)/p1p2...

for the first, say, N primes. Now, because we know it's irreducable (the top isn't divisible by any single prime, and the bottom is only divisible by the primes), we know that the decimal is only going to get larger (the bottom keeps increasing by more than an order of ten, and the top keeps increasing... so, for example, 3/10 is a shorter decimal than 111/1000). So as the limit tends to infinity, it needs to be irrational.

It's not rigorous, but that's probably how you would start it off
 
Last edited:
  • #4
The problem with that approach is making the transistion from arbitrarily large partial sums to the limit. For example, in the sum of a geometric series like 0.9+0.09+0.009+..., the common denominator/length of nonrepeating portion of the decimal expansion gets larger and larger as you take larger partial sums, but the limit is just 1.
 
  • #5
How do you know the top 'isn't divisble by any single prime'? It's certainly divisible by some primes, and there is nothing stopping it being one of the first N.
 
  • #6
matt grime said:
How do you know the top 'isn't divisble by any single prime'? It's certainly divisible by some primes, and there is nothing stopping it being one of the first N.

Looking back on it, that really is an error on my part, sorry :redface:

I got too ahead of myself
 
  • #7
Well, if Brun's constant were rational, you'd disprove both the twin primes conjecture and this conjecture.
 
  • #8
Dragonfall said:
Well, if Brun's constant were rational, you'd disprove both the twin primes conjecture and this conjecture.

You'd have to prove it rational first of course! It would disprove only one of twin primes and the one you have here, either one along with the others negation will work with Brun's constant being rational.

You can find a subset of the primes whose sum converges to 1 (or any other rational you like) without much difficulty given the fact that the full series diverges and the terms are tending to zero. Given a c>0 there are finite subsequences, p,...q, as far out as you'd need so that c<1/p+...+1/q<2c. Just use this to build a finite sequence whose sum is between 1-2^(-n) and 1, and repeat.
 
Last edited:
  • #9
shmoe said:
You can find a subset of the primes whose sum converges to 1 (or any other rational you like) without much difficulty given the fact that the full series diverges and the terms are tending to zero. Given a c>0 there are finite subsequences, p,...q, as far out as you'd need so that c<1/p+...+1/q<2c. Just use this to build a finite sequence whose sum is between 1-2^(-n) and 1, and repeat.

Actually, can't you just say:
Given some positive real number [itex]r>0[/tex] chose [itex]p_1,p_2...p_n...[/itex] where [itex]p_n[/itex] is the smallest prime such that:
[tex]\forall i < n, p_n \neq p_i[/itex]
and
[tex] \sum_{i=1}^{n} \frac{1}{p_n} < r[/tex]

Now, there will always be a suitable prime since the prior partial sum is strictly less than [itex]r[/itex], and the (non-empty) set of suitable primes will always have a unique least member.
 
Last edited:
  • #10
That's the first thing I thought of too, but wasn't immediately convinced a greedy algorithm would converge to your target, so I went another direction. It seemed likely that you could construct divergent series whose terms converged to zero yet this kind of greedy approach wouldn't always give a subsequence that works so I sort of expect some kind of density of the primes would be needed here. I haven't given it much thought. Can you prove this construction will converge to your r?
 
  • #11
shmoe said:
You'd have to prove it rational first of course! It would disprove only one of twin primes and the one you have here, either one along with the others negation will work with Brun's constant being rational.

You're right, I got my logic backwards.

Is there a strictly positive real number that no subseries of the harmonic series can converge to? If not then this 'conjecture' is false.
 
Last edited:
  • #12
I'm not sure what you mean Nate--I don't see how you extend your series to an infinite one. This may be close to what you were saying, or it may not. For the given r, define the sequence of primes pi by:
Code:
input r
r[sub]1[/sub] <-- r
i <-- 1
while true
   let p[sub]i[/sub] be the smallest prime not already chosen such that r[sub]i[/sub] - 1/p[sub]i[/sub] > 0
   r[sub]i+1[/sub] <-- r[sub]i[/sub] - 1/p[sub]i[/sub]
   i <-- i + 1
The pi here when their reciprocals are summed have to converge to r because by definition they can't exceed it, and they can't converge to something less than it. The latter is true because if they did converge to something less than r, say s, then let pn be some prime in the sequence, with n so large that r - s > 1/pn. Then the pi will also include all the consecutive primes after pn, up until a point where their reciprocal sum is greater than s (since the sum of the reciprocals of the primes diverges) but less than r (since 1/pj for j > n is always less than 1/pn, the first prime that puts the sum over s will not put it over r).
 
Last edited:
  • #13
That probably works for rationals, but are you sure you can get to things like, say, 1/sqrt(2) using that method?

EDIT: Wait,. it doesn't matter if you can. Whoops
 
  • #14
Yes, you can. r can be any positive real number. Actually, the same argument works for any infinite divergent series whose terms are positive and tend to 0.
 
Last edited:
  • #15
I concur. The conjecture's dead; 0rthodontist's greedy algorithm killed it.
 
  • #16
Excellent. Thanks for the input.
 
  • #17
0rthodontist said:
I'm not sure what you mean Nate--I don't see how you extend your series to an infinite one. This may be close to what you were saying, or it may not...

That's pretty much exactly what I was saying would work as an algorithm - and you've written out the proof that it works.
 

1. What is the sum of reciprocals of infinite subsets of primes?

The sum of reciprocals of infinite subsets of primes is a mathematical concept that involves adding all the reciprocals of an infinite number of prime numbers. This sum is known as the Euler's constant and is approximately equal to 0.5772156649.

2. How is the sum of reciprocals of infinite subsets of primes calculated?

The sum of reciprocals of infinite subsets of primes can be calculated using the Euler product formula, which states that the sum is equal to the reciprocal of the product of all prime numbers. This formula is represented as Π(1 - 1/p), where p represents all prime numbers.

3. What is the significance of the sum of reciprocals of infinite subsets of primes?

The sum of reciprocals of infinite subsets of primes has significant implications in number theory and has been used in various mathematical proofs. It is also closely related to the distribution of prime numbers and has been studied extensively by mathematicians.

4. Can the sum of reciprocals of infinite subsets of primes be infinite?

Yes, the sum of reciprocals of infinite subsets of primes can be infinite. This occurs when the prime numbers in the subset are chosen in a specific way, known as the divergent series. This is an important concept in mathematics, as it shows that even an infinite sum can still have an infinite value.

5. How does the sum of reciprocals of infinite subsets of primes relate to the Riemann zeta function?

The sum of reciprocals of infinite subsets of primes is closely related to the Riemann zeta function, which is a mathematical function that represents the sum of all reciprocals of positive integers. This function also plays a crucial role in the study of prime numbers and has connections to other areas of mathematics, such as complex analysis and number theory.

Similar threads

  • General Math
Replies
7
Views
1K
  • General Math
Replies
5
Views
2K
  • Set Theory, Logic, Probability, Statistics
Replies
22
Views
2K
Replies
4
Views
402
  • General Math
Replies
6
Views
1K
  • General Math
Replies
7
Views
485
Replies
1
Views
760
  • General Math
Replies
2
Views
736
Replies
15
Views
2K
Replies
13
Views
1K
Back
Top